Đến nội dung

Hình ảnh

Topic: [LTDH] Mỗi ngày hai bất đẳng thức.


  • Chủ đề bị khóa Chủ đề bị khóa
Chủ đề này có 215 trả lời

#21
tritanngo99

tritanngo99

    Đại úy

  • Điều hành viên THPT
  • 1644 Bài viết

Và dưới đây là lời giải bài 5 và bài 6:

Lời giải bài 5:

Áp dụng BĐT Cauchy ta có: $2+xy\ge \frac{3}{2}+\sqrt{2xy}=x^4+y^4+\frac{1}{xy}\ge 2x^2y^2+\frac{1}{xy}$(Cách chứng minh giống bạn MInhnguyenthe333).

Đặt $t=xy\implies t+2\ge 2t^2+\frac{1}{t}\implies (t+1)(t-1)(2t-1)\le 0\implies \frac{1}{2}\le t\le 1(\text{ (vì t>0)})$.

Khi đó: Áp dụng BĐT phụ quen thuộc: $\frac{1}{x^2+1}+\frac{1}{y^2+1}\le \frac{2}{xy+1}\forall xy\in[0;1]$.

Ta có: $L\le \frac{2}{xy+1}-\frac{3}{1+2xy}\implies L\le \frac{4}{t+1}-\frac{3}{2t+1}=\frac{5t+1}{2t^2+3t+1}$

$=\frac{7}{6}-\frac{(2t-1)(7t-1)}{2t^2+3t+1}\le \frac{7}{6}(\text{vì } \frac{1}{2}\le t)$.

Đẳng thức xảy ra khi $t=\frac{1}{2}$.

Vậy $L_{max}=\frac{7}{6}$ tại $x=y=\frac{1}{\sqrt{2}}$.

Lời giải bài 6:

Nhìn biểu thức $P$ ta nghĩ tới BĐT sau: $\frac{1}{a^2}+\frac{1}{b^2}\ge \frac{8}{(a+b)^2}$.

Áp dụng BĐT Cauchy ta có: $\frac{1}{a^2}+\frac{1}{b^2}\ge \frac{2}{ab}\ge \frac{8}{(a+b)^2}$

$\implies \frac{1}{(x+1)^2}+\frac{1}{(\frac{y}{2}+1)^2}\ge \frac{8}{(x+\frac{y}{2}+2)^2}$.

$\implies P\ge \frac{8}{(x+\frac{y}{2}+2)^2}+\frac{8}{(z+3)^2}\ge \frac{64}{(x+\frac{y}{2}+z+5)^2}$.

Ta có: $2x+y+2z=(2x+4y+2z)-3y\le (x^2+1)+(y^2+4)+(z^2+1)-3y\le 6$.

Suy ra: $P\ge \frac{64}{(3+5)^2}=1$.

Đẳng thức xảy ra khi và chỉ khi $x=1;y=2;z=1$.

Vậy $P_{min}=1$ tại $x=1,y=2,z=1.$


Bài viết đã được chỉnh sửa nội dung bởi tritanngo99: 10-08-2016 - 06:05


#22
tritanngo99

tritanngo99

    Đại úy

  • Điều hành viên THPT
  • 1644 Bài viết

Tiếp theo: 

Bài 7: Cho $a,b,c$ là các số thực dương thỏa mãn: $a^3+b^3+c^3=3$. Chứng minh rằng:

$\frac{a^3}{b^2-2b+3}+\frac{2b^3}{c^3+a^2-2a-3c+7}+\frac{3c^3}{a^4+b^4+a^2-2b^2-6a+11}\le \frac{3}{2}$

Bài 8:Cho $a,b,c$ là các số thực dương có tích bằng $1$. Chứng minh rằng:

$P=\frac{a^3(b^7+c+1)}{b^7(b+1)(c+1)}+\frac{b^3(c^7+a+1)}{c^7(c+1)(a+1)}+\frac{c^3(a^7+b+1)}{a^7(a+1)(b+1)}\ge \frac{9}{4}$



#23
Minhnguyenthe333

Minhnguyenthe333

    Trung úy

  • Thành viên
  • 804 Bài viết

Tiếp theo: 

Bài 7: Cho $a,b,c$ là các số thực dương thỏa mãn: $a^3+b^3+c^3=3$. Chứng minh rằng:

$\frac{a^3}{b^2-2b+3}+\frac{2b^3}{c^3+a^2-2a-3c+7}+\frac{3c^3}{a^4+b^4+a^2-2b^2-6a+11}\le \frac{3}{2}$

 

Ta xét mẫu số của 3 phân thức:

Dự đoán $a=b=c=1$ thì DBXR

Chú ý rằng: 

  $b^2-2b+3=(b-1)^2+2\geqslant 2$

  

  $c^3+a^2-2a-3c+7=(c-1)^2(c+2)+(a-1)^2+4\geqslant 4$

 

  $a^4+b^4+a^2-2b^2-6a+11=(a^2-1)^2+3(a-1)^2+(b^2-1)^2+6\geqslant 6$

 

$\Longrightarrow VT\leqslant \frac{a^3}{2}+\frac{2b^3}{4}+\frac{3c^3}{6}=\frac{a^3+b^3+c^3}{2}=\frac{3}{2}$

Dấu $"="$ xảy ra khi $a=b=c=1$



#24
tritanngo99

tritanngo99

    Đại úy

  • Điều hành viên THPT
  • 1644 Bài viết

Dưới đây là lời giải bài 7 và bài 8:

Lời giải bài 7: Mục tiêu là đánh giá mẫu thức phức tạp các phân thức:

Ta có: +)$\frac{a^3}{b^2-2b+3}=\frac{a^3}{(b-1)^2+2}\le \frac{a^3}{2}(1)$.

+) Xét hàm số: $f(c)=c^3+a^2-2a-3c+7$ trên $(0;+\infty)$ có: $f'(c)=3c^2-3=0$ tại $c=1$.

Lập bảng biến thiên hàm số suy ra $f(c)\ge f(1)=a^2-2a+5=(a-1)^2+4\ge 4$

$\implies \frac{2b^3}{c^3+a^2-2a-3c+7}\le \frac{b^3}{2}(2)$.

+Xét hàm số: $g(b)=a^4+b^4+a^2-2b^2-6a+11;g'(b)=0\iff b=1$.

Lập bảng biến thiên suy ra $g(b)\ge g(1)=a^4+a^2-6a+10=(a^2-1)^2+3(a-1)^2+6\ge 6$

$\implies \frac{3c^3}{a^4+b^4+a^2-2b^2-6a+11}\le \frac{c^3}{2}(3)$.

Từ $(1),(2),(3)$ suy ra $VT\le \frac{a^3+b^3+c^3}{2}=\frac{3}{2}$.

(Bài này có thể làm giống cách của bạn Minhnguyenthe333).

Lời giải bài 8: Hình thức khá khủng bố nên ta đi vào biến đổi mỗi phân thức:

Ta có: $P=\sum \frac{a^3(b^7+c+1)}{b^7(b+1)(c+1)}=\sum \frac{a^3}{(b+1)(c+1)}+\sum \frac{a^3}{b^7(b+1)}$.

Hướng đi quá quen thuộc, ta sẽ tách ra đi đánh giá $\sum \frac{a^3}{(b+1)(c+1)}$ và $\sum \frac{a^3}{b^7(b+1)}.$ rồi kết hợp lại:

Áp dụng BDT Cauchy ta có: $\sum \frac{a^3}{(b+1)(c+1)}+\sum \frac{b+1}{8}+\sum \frac{c+1}{8}\ge \frac{3}{4}(a+b+c)$.

$\implies \sum \frac{a^3}{(b+1)(c+1)}\ge \frac{1}{2}(a+b+c)-\frac{3}{4}(1)$.

Lại có: $\sum \frac{a^3}{b^7(b+1)}+\sum \frac{b+1}{4}\ge \sum \sqrt{\frac{a^3}{b^7}}\ge 3\sqrt[6]{\frac{a^3b^3c^3}{b^7c^7a^7}}=3(\text{ vì abc=1})$.

Suy ra $\sum \frac{a^3}{b^7(b+1)}\ge \frac{9}{4}-\frac{1}{4}(a+b+c)(2)$.

Từ $(1),(2)$ suy ra $P\ge \frac{1}{2}(a+b+c)-\frac{3}{4}+\frac{9}{4}-\frac{1}{4}(a+b+c)=\frac{1}{4}(a+b+c)+\frac{3}{2}\ge \frac{3}{4}+\frac{3}{2}=\frac{9}{4}.$.

Ps: Bài làm tuy "khủng" nhưng hướng đi hoàn toàn tự nhiên và khá nhẹ nhàng.



#25
tritanngo99

tritanngo99

    Đại úy

  • Điều hành viên THPT
  • 1644 Bài viết

Tiếp theo:

Bài 9: cho $a,b,c$ là các số thực không âm có tổng bằng 1. Tìm giá trị nhỏ nhất của biểu thức:

$P=\frac{a}{1+b^2+c^2}+\frac{b}{1+a^2+c^2}+\frac{c}{1+a^2+b^2}$.

Bài 10:Giả sử $a,b,c$ là các số thực không âm. Chứng minh bất đẳng thức:

$\frac{1}{a^2+b^2}+\frac{1}{b^2+c^2}+\frac{1}{c^2+a^2}\ge \frac{10}{(a+b+c)^2}$.


Bài viết đã được chỉnh sửa nội dung bởi tritanngo99: 11-08-2016 - 06:02


#26
tritanngo99

tritanngo99

    Đại úy

  • Điều hành viên THPT
  • 1644 Bài viết

Dưới đây là lời giải bài 9 và bài 10:

Lời giải bài 9: Sử dụng bất đẳng thức $Cauchy-Schwarz$ ta có:

Đặt $P=a(1+b^2+c^2)+b(1+a^2+c^2)+c(1+a^2+b^2)$.

$(\frac{a}{1+b^2+c^2})+\frac{b}{1+a^2+c^2}+\frac{c}{1+a^2+b^2})P\ge (a+b+c)^2$.

Xét biểu thức: $S_{a,b,c}=ab(a+b)+bc(b+c)+ca(c+a)$.

Ta sẽ chứng minh $S_{a,b,c}\le \frac{1}{4}$. Thật vậy không mất tính tổng quát giả sử $c\ge b\ge a$.

Xét biểu thức: $S_{a,b,c}-S_{a+b,c,0}=a^2(b+c)+b^2(c+a)+c^2(a+b)-(a+b)^2c-c^2(a+b)=ab(a+b-2c)\le 0$.

Mặt khác $S_{a+b,c,0}=(a+b)c(a+b+c)=(a+b)c\le \frac{1}{4}(a+b+c)^2=\frac{1}{4}$.

Như vậy: $P=a+b+c+S_{a,b,c}\le \frac{5}{4}$. Suy ra:

$\frac{a}{1+b^2+c^2}+\frac{b}{1+a^2+c^2}+\frac{c}{1+a^2+b^2}\ge \frac{4}{5}$.

Đẳng thức xảy ra khi và chỉ khi $b=c=\frac{1}{2};a=0$ và các hoán vị.

Lời giải bài 10: Giả sử $c=min(a,b,c)$. Dễ thấy rằng:

$b^2+c^2\le (b+\frac{c}{2})^2,a^2+c^2\le (c+\frac{a}{2})^2,a^2+b^2\le (a+\frac{c}{2})^2+(b+\frac{c}{2})^2$.

Vậy ta có: $VT\ge \frac{1}{(a+\frac{c}{2})^2}+\frac{1}{(b+\frac{c}{2})^2}+\frac{1}{(a+\frac{c}{2})^2+(b+\frac{c}{2})^2}\rightarrow Q$.

Áp dụng BDT quen thuộc: $\frac{1}{A^2}+\frac{1}{B^2}\ge \frac{8}{(A+B)^2}$.

Suy ra: $Q=\frac{3}{4}[\frac{1}{(a+\frac{c}{2})^2}+\frac{1}{(b+\frac{c}{2})^2}]+\frac{1}{4}[\frac{1}{(a+\frac{c}{2})^2}+\frac{1}{(b+\frac{c}{2})^2}]+\frac{1}{(a+\frac{c}{2})^2+(b+\frac{c}{2})^2}$.

$\ge \frac{6}{(a+b+c)^2}+\frac{1}{2(a+\frac{c}{2})(b+\frac{c}{2})}+\frac{1}{(a+\frac{c}{2})^2+(b+\frac{c}{2})^2}\ge \frac{10}{(a+b+c)^2}(dpcm)$.

Đẳng thức xảy ra tại khi và chỉ khi $a=b,c=0$ hoặc các hoán vị.


Bài viết đã được chỉnh sửa nội dung bởi tritanngo99: 12-08-2016 - 05:41


#27
tritanngo99

tritanngo99

    Đại úy

  • Điều hành viên THPT
  • 1644 Bài viết

Tiếp theo: 

Bài 11: Chứng minh rằng với các số dương $a,b,c$ và $abc=8$ ta có bất đẳng thức sau:

$\frac{a^2}{\sqrt{(1+a^3)(1+b^3)}}+\frac{b^2}{\sqrt{(1+b^3)(1+c^3)}}+\frac{c^2}{\sqrt{(1+c^3)(1+a^3)}}\ge \frac{4}{3}$.

Bài 12: Cho $x,y,z\in [-1;1]$ và $x+y+z=0$. Chứng minh bất đẳng thức:

$\sqrt{1+x+y^2}+\sqrt{1+y+z^2}+\sqrt{1+z+x^2}\ge 3$



#28
VODANH9X

VODANH9X

    Trung sĩ

  • Thành viên
  • 114 Bài viết

Tiếp theo: 

Bài 12: Cho $x,y,z\in [-1;1]$ và $x+y+z=0$. Chứng minh bất đẳng thức:

$\sqrt{1+x+y^2}+\sqrt{1+y+z^2}+\sqrt{1+z+x^2}\ge 3$

Ta áp dụng bđt sau:khi $ab\geq 0$ ta có $\sqrt{1+a}+\sqrt{1+b}\geq 1+\sqrt{1+a+b}$

Cái này bình phương biến đổi tương đương là được.

Chú ý rằng trong $x+y^{2},y+z^{2},z+x^{2}$ có ít nhất 2 số cùng dấu.

Không mất tính tổng quát ta giả sử $(x+y^{2})(y+z^{2})\geq 0$

Áp dụng bđt trên ta được:

$\sqrt{1+x+y^{2}}+\sqrt{1+y+z^{2}}+\sqrt{1+z+x^{2}}$

$\geq 1+\sqrt{1+x+y^{2}+y+z^{2}}+\sqrt{1+z+x^{2}}$

$=1+\sqrt{(\sqrt{1-z+z^{2}})^{2}+y^{2}}+\sqrt{(\sqrt{1+z})^{2}+x^{2}}$

$\geq 1+ \sqrt{(\sqrt{1-z+z^{2}}+\sqrt{1+z})^{2}+(x+y)^{2}}$  (bđt Mincopxki)

$= \sqrt{(\sqrt{1-z+z^{2}}+\sqrt{1+z})^{2}+z^{2}}$

Ta cần chứng minh $(\sqrt{1-z+z^{2}}+\sqrt{1+z})^{2}+z^{2}\geq 4$

$\Leftrightarrow 2z^{2}+2\sqrt{1+z^{3}}\geq 2\Leftrightarrow z^{2}(2-z)(z+1)\geq 0$

Luôn đúng vì $-1\leq x,y,z\leq 1$

Suy ra đpcm.



#29
cyndaquil

cyndaquil

    Hạ sĩ

  • Thành viên
  • 63 Bài viết

Tiếp theo: 

Bài 11: Chứng minh rằng với các số dương $a,b,c$ và $abc=8$ ta có bất đẳng thức sau:

$\frac{a^2}{\sqrt{(1+a^3)(1+b^3)}}+\frac{b^2}{\sqrt{(1+b^3)(1+c^3)}}+\frac{c^2}{\sqrt{(1+c^3)(1+a^3)}}\ge \frac{4}{3}$.

Bài 12: Cho $x,y,z\in [-1;1]$ và $x+y+z=0$. Chứng minh bất đẳng thức:

$\sqrt{1+x+y^2}+\sqrt{1+y+z^2}+\sqrt{1+z+x^2}\ge 3$

Ta có $\sqrt{a^3+1}=\sqrt{(a+1)(a^2-a+1)} \le \frac{a^2+2}{2}$
cmtt cho $b,c$
Suy ra $VT \ge \sum\frac{4a^2}{(a^2+2)(b^2+2)}$
Nên ta chỉ cần cm $\sum\frac{a^2}{(a^2+2)(b^2+2)} \ge \frac 13(\star)$ là đủ
$(\star) \Leftrightarrow \frac{\sum a^2b^2+2(a^2+b^2+c^2)}{(a^2+2)(b^2+2)(c^2+2)} \ge \frac 13$
$\Leftrightarrow 3\sum a^2b^2+6(a^2+b^2+c^2) \ge a^2b^2c^2+2 \sum a^2b^2+4(a^2+b^2+c^2)+8$
$\Leftrightarrow (a^2b^2+b^2c^2+c^2a^2)+2(a^2+b^2+c^2) \ge 72$
Dễ dàng cm bdt này với đk $abc=8$


#30
tritanngo99

tritanngo99

    Đại úy

  • Điều hành viên THPT
  • 1644 Bài viết

Lời giải của hai bài toán 11bài toán 12 hoàn toàn giống lời giải của bạn VODANH9Xcyldaquil. Nên mình xin đề xuất hai bài tiếp theo:

Bài 13: Tìm GTLN,GTNN của biểu thức:

$Q=\frac{\sqrt{3}}{4}(x+y+xy)+\frac{\sqrt{6}}{6}\sqrt{x^2+y^2-xy}$ biết $x,y$ thỏa mãn: $x,y\in (0,1]$ và $x+y=3xy$.

Bài 14: Cho $a,b,c$ là các số không âm thỏa mãn: $a>b>c$ và $3ab+5bc+7ca\le 9$. Tìm GTNN của biểu thức:

$P=\frac{32}{(a-b)^4}+\frac{1}{(b-c)^4}+\frac{1}{(c-a)^4}$. 

 



#31
VODANH9X

VODANH9X

    Trung sĩ

  • Thành viên
  • 114 Bài viết

Lời giải của hai bài toán 11bài toán 12 hoàn toàn giống lời giải của bạn VODANH9Xcyldaquil. Nên mình xin đề xuất hai bài tiếp theo:

Bài 13: Tìm GTLN,GTNN của biểu thức:

$Q=\frac{\sqrt{3}}{4}(x+y+xy)+\frac{\sqrt{6}}{6}\sqrt{x^2+y^2-xy}$ biết $x,y$ thỏa mãn: $x,y\in (0,1]$ và $x+y=3xy$.

Bài 14: Cho $a,b,c$ là các số không âm thỏa mãn: $a>b>c$ và $3ab+5bc+7ca\le 9$. Tìm GTNN của biểu thức:

$P=\frac{32}{(a-b)^4}+\frac{1}{(b-c)^4}+\frac{1}{(c-a)^4}$. 

Bài 14:Vì $a> b> c\geq 0$ nên $3ab\leq 3ab+5bc+7ac\leq 9\Rightarrow ab\leq 3$

P$\geq \frac{32}{(a-b)^{4}}+\frac{1}{a^{4}}+\frac{1}{b^{4}}$

$\Rightarrow P\geq \frac{a^{2}b^{2}}{9}(\frac{32}{(a-b)^{4}}+\frac{1}{a^{4}}+\frac{1}{b^{4}})$

$=\frac{1}{9}(\frac{32a^{2}b^{2}}{a^{2}-2ab+b^{2}}+\frac{a^{2}}{b^{2}}+\frac{b^{2}}{a^{2}})$

$=\frac{1}{9}(\frac{32}{(\frac{a}{b}+\frac{b}{a}-2)^{2}}+(\frac{a}{b}+\frac{b}{a})^{2}-2)$

Xét hàm số $f(t)=\frac{1}{9}(\frac{32}{(t-2)^{2}}+t^{2}-2)$ với $t=\frac{a}{b}+\frac{b}{a}$,$t> 2$

$f(t)'=0\Leftrightarrow t=4$

Lập bảng biến thiên suy ra $P\geq f(4)=\frac{10}{9}$

Vậy Min $P=\frac{10}{9}$ dấu bằng xảy ra khi $ab=3 và \frac{a}{b}+\frac{b}{a}=4$



#32
VODANH9X

VODANH9X

    Trung sĩ

  • Thành viên
  • 114 Bài viết

Lời giải của hai bài toán 11bài toán 12 hoàn toàn giống lời giải của bạn VODANH9Xcyldaquil. Nên mình xin đề xuất hai bài tiếp theo:

Bài 13: Tìm GTLN,GTNN của biểu thức:

$Q=\frac{\sqrt{3}}{4}(x+y+xy)+\frac{\sqrt{6}}{6}\sqrt{x^2+y^2-xy}$ biết $x,y$ thỏa mãn: $x,y\in (0,1]$ và $x+y=3xy$.

Bài 14: Cho $a,b,c$ là các số không âm thỏa mãn: $a>b>c$ và $3ab+5bc+7ca\le 9$. Tìm GTNN của biểu thức:

$P=\frac{32}{(a-b)^4}+\frac{1}{(b-c)^4}+\frac{1}{(c-a)^4}$. 

Bài 13:

Đặt $x+y=t\Rightarrow xy=\frac{t}{3}$

$\Rightarrow Q=\frac{\sqrt{3}}{3}t+\frac{\sqrt{6}}{6}\sqrt{t^{2}-t}$

$x+y=3xy\leq \frac{3}{4}(x+y)^{2}\Rightarrow x+y\geq \frac{4}{3}\Leftrightarrow t\geq \frac{4}{3}$

Ta lại có $(x-1)(y-1)\geq 0\Leftrightarrow xy+1\geq x+y\Leftrightarrow \frac{t}{3}+1\geq t\Leftrightarrow t\leq \frac{3}{2}$

Xét hàm $f(t)=\frac{\sqrt{3}}{3}t+\frac{\sqrt{6}}{6}\sqrt{t^{2}-t}$ trên $\frac{4}{3}\leq t\leq \frac{3}{2}$

Hàm này đồng biến trên đoạn đó.

Vậy Min $Q=f(\frac{4}{3})$ khi $x+y=3xy,x+y=\frac{4}{3} \Leftrightarrow x=y=\frac{2}{3}$

       Max$Q=f(\frac{3}{2})$ khi $x+y=3xy,x+y=\frac{3}{2} \Leftrightarrow x=1,y=\frac{1}{2}$,và ngược lại.



#33
tritanngo99

tritanngo99

    Đại úy

  • Điều hành viên THPT
  • 1644 Bài viết

Dưới đây là lời giải bài 13 và bài 14:

Lời giải bài 13: 

Đặt $x=b-c;y=a-c$ ta có: $P=\frac{32}{(x-y)^4}+\frac{1}{x^4}+\frac{1}{y^4}$.

Ta thấy: $3ab\le 3ab+5bc+7ca\le 9\implies xy\le ab\le 3$.

Khi đó: $9P=\frac{32*9}{(x-y)^4}+\frac{9}{x^4}+\frac{9}{y^4}\ge \frac{32(xy)^2}{(x^2-y^2-2xy)^2}+\frac{(xy)^2}{x^4}+\frac{(xy)^2}{y^4}$

$=\frac{32}{(\frac{x}{y}+\frac{y}{x}-2)^2}+(\frac{x}{y})^2+(\frac{y}{x})^2$.

Đặt $t=\frac{x}{y}+\frac{y}{x}(t\ge 2)$. Ta có: $9P=\frac{32}{(t-2)^2}+t^2-2=f(t)$.

$f'(t)=2t-\frac{64}{(t-2)^3}$.

$f'(t)=0\iff t^4-6t^3+12t^2-8t-32=0\iff (t-4)(t^3-2t^2+4t+8)=0\iff t=4(t\ge 2)$.

Lập BBT ta có: $P_{min}=\frac{22}{9}$. Dấu $=$ xảy ra khi $\left\{\begin{matrix} x^2+y^2=4xy\\ xy=3 \end{matrix}\right.$

$\iff (x;y)=(\frac{\sqrt{6}+3\sqrt{2}}{2};\frac{\sqrt{6}-3\sqrt{2}}{2})$ và ngược lại. 

Hay $(a;b;c)=(\frac{\sqrt{6}+3\sqrt{2}}{2};\frac{\sqrt{6}-3\sqrt{2}}{2};0)\text{ do a>b>c}$

Lời giải bài 14: Ý tưởng giống bài bạn VODANH9X


Bài viết đã được chỉnh sửa nội dung bởi tritanngo99: 14-08-2016 - 05:13


#34
tritanngo99

tritanngo99

    Đại úy

  • Điều hành viên THPT
  • 1644 Bài viết

Tiếp theo:

Bài 15: Cho $a,b,c>0$. Tìm giá trị nhỏ nhất của biểu thức:$P=\frac{4a^3+3b^3+2c^3-3b^2c}{(a+b+c)^3}$

Bài 16: Cho $x,y$ là các số thực thỏa mãn điều kiện: $x+y=2\sqrt{x+2}+3\sqrt{y-2014}+2012$. Tìm GTNN và GTLN của biểu thức:

$S=(x-1)^2+(y-1)^2+\frac{2015+2xy\sqrt{x+y+1}}{\sqrt{x+y+1}}$



#35
nguyenhongsonk612

nguyenhongsonk612

    Thượng úy

  • Thành viên
  • 1451 Bài viết

 

Bài 16: Cho $x,y$ là các số thực thỏa mãn điều kiện: $x+y=2\sqrt{x+2}+3\sqrt{y-2014}+2012$. Tìm GTNN và GTLN của biểu thức:

$S=(x-1)^2+(y-1)^2+\frac{2015+2xy\sqrt{x+y+1}}{\sqrt{x+y+1}}$

Đk: $x\geqslant -2; y\geqslant 2014$

Từ GT $\Rightarrow x+y\geqslant 2012$

Áp dụng BĐT Bunhiacopxki ta được 

$x+y-2012=2\sqrt{x+2}+3\sqrt{y-2014}\leqslant \sqrt{13(x+y-2012)}\Leftrightarrow x+y\leqslant 2025$

Vậy ta được $2012\leqslant x+y\leqslant 2025$

$S=(x+y)^2-2(x+y)+2+\frac{2015}{\sqrt{x+y+1}}=(x+y-1)^2+1+\frac{2015}{\sqrt{x+y+1}}$

Đặt $t=\sqrt{x+y+1}(\sqrt{2013}\leqslant t \leqslant \sqrt{2026})$

$\Rightarrow S=f(t)=(t^2-2)^2+1+\frac{2015}{t}$

Khảo sát hàm $f(t)$ trên $\left [ \sqrt{2013};\sqrt{2026} \right ]$ ta được 

$\min f(t)=f\left ( \sqrt{2013} \right )=4044122+\frac{2015}{\sqrt{2013}}\Leftrightarrow x=-2;y=2014$

$\max f(t)=f\left ( \sqrt{2026} \right )=4096577+\frac{2015}{\sqrt{2026}}$$\Leftrightarrow x=2;y=2023$


Bài viết đã được chỉnh sửa nội dung bởi nguyenhongsonk612: 14-08-2016 - 15:25

"...Từ ngay ngày hôm nay tôi sẽ chăm chỉ học hành như Stardi, với đôi tay nắm chặt và hàm răng nghiến lại đầy quyết tâm. Tôi sẽ nỗ lực với toàn bộ trái tim và sức mạnh để hạ gục cơn buồn ngủ vào mỗi tối và thức dậy sớm vào mỗi sáng. Tôi sẽ vắt óc ra mà học và không nhân nhượng với sự lười biếng. Tôi có thể học đến phát bệnh miễn là thoát khỏi cuộc sống nhàm chán khiến mọi người và cả chính tôi mệt mỏi như thế này. Dũng cảm lên! Hãy bắt tay vào công việc với tất cả trái tim và khối óc. Làm việc để lấy lại niềm vui, lấy lại nụ cười trên môi thầy giáo và cái hôn chúc phúc của bố tôi. " (Trích "Những tấm lòng cao cả")

~O) 


#36
tpdtthltvp

tpdtthltvp

    Trung úy

  • Điều hành viên THCS
  • 831 Bài viết

Bài 15: Cho $a,b,c>0$. Tìm giá trị nhỏ nhất của biểu thức:$P=\frac{4a^3+3b^3+2c^3-3b^2c}{(a+b+c)^3}$

 

Áp dụng bất đẳng thức $AM-GM:$

$$2b^3+c^3=b^3+b^3+c^3\geq 3b^2c$$

Do đó:

$$P\geq \frac{4a^3+b^3+c^3}{(a+b+c)^3}$$

Theo BĐT $Holder,$ ta có:

$$(4a^3+b^3+c^3)(\frac{1}{2}+1+1)(\frac{1}{2}+1+1)\geq (a+b+c)^3\Leftrightarrow \frac{4a^3+b^3+c^3}{(a+b+c)^3}\geq \frac{4}{25}$$

Vậy $\min P=\frac{4}{25}\Leftrightarrow a=\frac{1}{2}b=\frac{1}{2}c$


$\color{red}{\mathrm{\text{How I wish I could recollect, of circle roud}}}$

$\color{red}{\mathrm{\text{The exact relation Archimede unwound ! }}}$

 


#37
tritanngo99

tritanngo99

    Đại úy

  • Điều hành viên THPT
  • 1644 Bài viết

Lời giải bài 15bài 16 giống hoàn toàn lời giải bài bạn nguyenhongsonk612tpdtthltvp nên mình đề xuất hai bài tiếp theo như sau:

Bài 17: Cho các số thực $x,y,z$ thay đổi thỏa mãn: $x^2+y^2+z^2=1$. Tìm GTNN của biểu thức:

$P=(xy+yz+2zx)^2-\frac{8}{(x+y+z)^2-xy-yz+2}$

Bài 18: Cho $x,y,z$ là ba số thực thỏa mãn: $2x+3y+z=40$. Tìm GTNN của biểu thức:

$S=2\sqrt{x^2+1}+3\sqrt{y^2+16}+\sqrt{z^2+36}$ 



#38
NTA1907

NTA1907

    Thượng úy

  • Thành viên
  • 1014 Bài viết

Bài 18: Cho $x,y,z$ là ba số thực thỏa mãn: $2x+3y+z=40$. Tìm GTNN của biểu thức:

$S=2\sqrt{x^2+1}+3\sqrt{y^2+16}+\sqrt{z^2+36}$ 

Áp dụng bất đẳng thức Min-cốp-xki ta có:

$S=\sqrt{4x^{2}+4}+\sqrt{9y^{2}+144}+\sqrt{z^{2}+36}\geq \sqrt{(2x+3y+z)^{2}+(2+12+6)^{2}}=20\sqrt{5}$

Dấu = xảy ra$\Leftrightarrow x=2, y=8, z=12$


Vũ trụ không có biên trong không gian, không có bắt đầu và kết thúc trong thời gian và chẳng có việc gì cho đấng sáng thế phải làm ở đây cả.

 


#39
royal1534

royal1534

    Trung úy

  • Điều hành viên THCS
  • 773 Bài viết

Lời giải bài 15bài 16 giống hoàn toàn lời giải bài bạn nguyenhongsonk612tpdtthltvp nên mình đề xuất hai bài tiếp theo như sau:

Bài 17: Cho các số thực $x,y,z$ thay đổi thỏa mãn: $x^2+y^2+z^2=1$. Tìm GTNN của biểu thức:

$P=(xy+yz+2zx)^2-\frac{8}{(x+y+z)^2-xy-yz+2}$

Bài 18: Cho $x,y,z$ là ba số thực thỏa mãn: $2x+3y+z=40$. Tìm GTNN của biểu thức:

$S=2\sqrt{x^2+1}+3\sqrt{y^2+16}+\sqrt{z^2+36}$ 

Thấy mọi người giải nhanh quá nên không kịp giải :D

--------

Bài 17:

Sử dụng giả thiết $x^2+y^2+z^2=1$. Ta viết lại $P=(xy+yz+2zx)^2-\frac{8}{(x+y+z)^2-xy-yz+2}=(xy+yz+2zx)^2-\frac{8}{3+xy+yz+2zx}$

Bây giờ ta chỉ cần tìm Min của $xy+yz+2zx$ là bài toán được giải quyết

Hiển nhiên ta có:

-$(x+y+z)^2 \geq 0$

-$2zx \geq -(x^2+z^2)=-(1-y^2)=y^2-1$

$\Rightarrow 2xy+2yz+4zx+1 \geq y^2-1$

$\Leftrightarrow 2xy+2yz+4zx \geq y^2-2 \geq -2$

$\Leftrightarrow xy+yz+2zx \geq -1$

------

Dự đoán $MinP=(-3)$ ta đi chứng minh 

 $P=(xy+yz+2zx)^2-\frac{8}{3+xy+yz+2zx} \geq -3$

$\Leftrightarrow t^2-\frac{8}{3+t} \geq -3$ (Với $t =xy+yz+2zx \geq -1$)

$\Leftrightarrow (t+1)^3 \geq 0$: Đúng vì $t \geq -1$

Dấu '=' xảy ra khi bộ $(x,y,z)$ thõa: $x=-z,y=0,x^2+y^2+z^2=1$


Bài viết đã được chỉnh sửa nội dung bởi royal1534: 15-08-2016 - 23:11


#40
thoai6cthcstqp

thoai6cthcstqp

    Trung sĩ

  • Thành viên
  • 146 Bài viết

Lời giải bài 15bài 16 giống hoàn toàn lời giải bài bạn nguyenhongsonk612tpdtthltvp nên mình đề xuất hai bài tiếp theo như sau:
Bài 17: Cho các số thực $x,y,z$ thay đổi thỏa mãn: $x^2+y^2+z^2=1$. Tìm GTNN của biểu thức:
$P=(xy+yz+2zx)^2-\frac{8}{(x+y+z)^2-xy-yz+2}$
Bài 18: Cho $x,y,z$ là ba số thực thỏa mãn: $2x+3y+z=40$. Tìm GTNN của biểu thức:
$S=2\sqrt{x^2+1}+3\sqrt{y^2+16}+\sqrt{z^2+36}$

Bài 18: Ta có các đánh giá sau:
$2\sqrt{x^2+1}\geq $$\frac{4\sqrt{5}x}{5}+\frac{2\sqrt{5}}{5}$
$3\sqrt{y^2+16}\geq $$\frac{6\sqrt{5}y}{5}+\frac{12\sqrt{5}}{5}$
$\sqrt{z^2+36}\geq $$\frac{2\sqrt{5}x}{5}+\frac{6\sqrt{5}}{5}$.
Công việc còn lại rất đơn giản.

Bài viết đã được chỉnh sửa nội dung bởi thoai6cthcstqp: 15-08-2016 - 21:37

Cá mỏ nhọn <3





0 người đang xem chủ đề

0 thành viên, 0 khách, 0 thành viên ẩn danh